Find maximum value under constraints by Lagrange multipliers or another method












0












$begingroup$


Let $f(x,y)=(x + 2y)^2 + (3x + 4y)^2.$



Then for $(x,y)$ on the circle $x^2+y^2=1 $ what is the maximum value of $f$?



I tried to use Lagrange multiplier method with the constraint $g(x,y)= x^2 + y^2 =1.$



However, now I realize that this way is so complicated on this problem
($lambda = 15 pm sqrt{221}$)



How can I solve this problem?










share|cite|improve this question











$endgroup$












  • $begingroup$
    Did you try using polar coordinates?
    $endgroup$
    – Damien
    Jan 11 at 6:09
















0












$begingroup$


Let $f(x,y)=(x + 2y)^2 + (3x + 4y)^2.$



Then for $(x,y)$ on the circle $x^2+y^2=1 $ what is the maximum value of $f$?



I tried to use Lagrange multiplier method with the constraint $g(x,y)= x^2 + y^2 =1.$



However, now I realize that this way is so complicated on this problem
($lambda = 15 pm sqrt{221}$)



How can I solve this problem?










share|cite|improve this question











$endgroup$












  • $begingroup$
    Did you try using polar coordinates?
    $endgroup$
    – Damien
    Jan 11 at 6:09














0












0








0





$begingroup$


Let $f(x,y)=(x + 2y)^2 + (3x + 4y)^2.$



Then for $(x,y)$ on the circle $x^2+y^2=1 $ what is the maximum value of $f$?



I tried to use Lagrange multiplier method with the constraint $g(x,y)= x^2 + y^2 =1.$



However, now I realize that this way is so complicated on this problem
($lambda = 15 pm sqrt{221}$)



How can I solve this problem?










share|cite|improve this question











$endgroup$




Let $f(x,y)=(x + 2y)^2 + (3x + 4y)^2.$



Then for $(x,y)$ on the circle $x^2+y^2=1 $ what is the maximum value of $f$?



I tried to use Lagrange multiplier method with the constraint $g(x,y)= x^2 + y^2 =1.$



However, now I realize that this way is so complicated on this problem
($lambda = 15 pm sqrt{221}$)



How can I solve this problem?







lagrange-multiplier






share|cite|improve this question















share|cite|improve this question













share|cite|improve this question




share|cite|improve this question








edited Jan 11 at 6:14









David G. Stork

10.7k31332




10.7k31332










asked Jan 11 at 5:55









hewhew

306




306












  • $begingroup$
    Did you try using polar coordinates?
    $endgroup$
    – Damien
    Jan 11 at 6:09


















  • $begingroup$
    Did you try using polar coordinates?
    $endgroup$
    – Damien
    Jan 11 at 6:09
















$begingroup$
Did you try using polar coordinates?
$endgroup$
– Damien
Jan 11 at 6:09




$begingroup$
Did you try using polar coordinates?
$endgroup$
– Damien
Jan 11 at 6:09










5 Answers
5






active

oldest

votes


















2












$begingroup$

If Lagrange's is not mandatory, WLOG choose $x=cos t, y=sin t$ to find



$$f(x,y)=Acos^2t+Bsin tcos t+Csin^2t=g(t)text{(say)}$$



Use $cos2t=1-2sin^2t=2cos^2t-1,sin2t=2sin tcos t$ to find $g(t)$ to be of the form $$pcos2t+qsin2t+r$$



Now use $-sqrt{p^2+q^2}le pcos2t+qsin2tlesqrt{p^2+q^2}$






share|cite|improve this answer









$endgroup$













  • $begingroup$
    Thank you very much!!
    $endgroup$
    – hew
    Jan 11 at 6:35



















1












$begingroup$

This is simple enough that you can use your constraint as a substitution, i.e., $y = sqrt{1 - x^2}$.



Then your function is:



$$f(x) = left(2 sqrt{1-x^2}+xright)^2+left(4 sqrt{1-x^2}+3 xright)^2$$



enter image description here



Its derivative is:



$${d f(x) over dx} = -frac{4 left(14 x^2+5 sqrt{1-x^2} x-7right)}{sqrt{1-x^2}}$$



Set it to zero and find:



$$x = left{-sqrt{frac{1}{2}+frac{5}{2 sqrt{221}}},sqrt{frac{1}{442} left(221-5 sqrt{221}right)}right} approx { -0.817416, 0.576048 }$$



where clearly one is a minimum, the other a maximum.



For "culture," here is the figure in three dimensions:



enter image description here






share|cite|improve this answer











$endgroup$













  • $begingroup$
    Thank you very much!!
    $endgroup$
    – hew
    Jan 11 at 6:34










  • $begingroup$
    @hew: So perhaps a (+1) is in order?
    $endgroup$
    – David G. Stork
    Jan 11 at 6:36



















0












$begingroup$

Exact solution with Lagrange multiplier method is:
$$f_{min}=15-sqrt{221}approx0.134$$
if
$$x=frac{sqrt{5 sqrt{221}+221}}{sqrt{442}}approx0.817,;
y=-frac{sqrt{221-5 sqrt{221}}}{sqrt{442}}approx-0.576$$

or
$$x=-frac{sqrt{5 sqrt{221}+221}}{sqrt{442}}approx-0.817,;
y=frac{sqrt{221-5 sqrt{221}}}{sqrt{442}}approx0.576$$

enter image description here






share|cite|improve this answer









$endgroup$





















    0












    $begingroup$

    It is possible to find the solution by using the polar coordinates:



    $$f(theta) = 10cos^2theta + 20sin^2theta + 28sinthetacostheta = 10 + 10sin^2theta + 14sin2theta$$



    The extrema are found by deriving:



    $$f'(theta) = 20sintheta costheta + 28cos2theta = 10sin2theta + 28cos2theta = 0$$



    And we get
    $$theta = -frac{1}{2} arctanfrac{14}{5} + kfrac{pi}{2} $$
    where $k$ is any integer between $0$ and $3$



    We can check numerically that the maximum corresponds to
    $$theta = -frac{1}{2} arctanfrac{14}{5} + kpi approx -0.6138 + kpi$$



    where $k$ is equal to $0$ or $1$



    This corresponds to $x approx 0.81754, ,y approx -0.576$ and $x approx -0.81754, ,y approx 0.576$






    share|cite|improve this answer











    $endgroup$





















      0












      $begingroup$

      The maximum value of a quadratic form on the unit circle is equal to its largest eigenvalue. $f$ simplifies to $10x^2+28xy+20y^2$ and so the eigenvalues of $f$ are the roots of $lambda^2-30lambda+4$.






      share|cite|improve this answer









      $endgroup$













        Your Answer





        StackExchange.ifUsing("editor", function () {
        return StackExchange.using("mathjaxEditing", function () {
        StackExchange.MarkdownEditor.creationCallbacks.add(function (editor, postfix) {
        StackExchange.mathjaxEditing.prepareWmdForMathJax(editor, postfix, [["$", "$"], ["\\(","\\)"]]);
        });
        });
        }, "mathjax-editing");

        StackExchange.ready(function() {
        var channelOptions = {
        tags: "".split(" "),
        id: "69"
        };
        initTagRenderer("".split(" "), "".split(" "), channelOptions);

        StackExchange.using("externalEditor", function() {
        // Have to fire editor after snippets, if snippets enabled
        if (StackExchange.settings.snippets.snippetsEnabled) {
        StackExchange.using("snippets", function() {
        createEditor();
        });
        }
        else {
        createEditor();
        }
        });

        function createEditor() {
        StackExchange.prepareEditor({
        heartbeatType: 'answer',
        autoActivateHeartbeat: false,
        convertImagesToLinks: true,
        noModals: true,
        showLowRepImageUploadWarning: true,
        reputationToPostImages: 10,
        bindNavPrevention: true,
        postfix: "",
        imageUploader: {
        brandingHtml: "Powered by u003ca class="icon-imgur-white" href="https://imgur.com/"u003eu003c/au003e",
        contentPolicyHtml: "User contributions licensed under u003ca href="https://creativecommons.org/licenses/by-sa/3.0/"u003ecc by-sa 3.0 with attribution requiredu003c/au003e u003ca href="https://stackoverflow.com/legal/content-policy"u003e(content policy)u003c/au003e",
        allowUrls: true
        },
        noCode: true, onDemand: true,
        discardSelector: ".discard-answer"
        ,immediatelyShowMarkdownHelp:true
        });


        }
        });














        draft saved

        draft discarded


















        StackExchange.ready(
        function () {
        StackExchange.openid.initPostLogin('.new-post-login', 'https%3a%2f%2fmath.stackexchange.com%2fquestions%2f3069516%2ffind-maximum-value-under-constraints-by-lagrange-multipliers-or-another-method%23new-answer', 'question_page');
        }
        );

        Post as a guest















        Required, but never shown

























        5 Answers
        5






        active

        oldest

        votes








        5 Answers
        5






        active

        oldest

        votes









        active

        oldest

        votes






        active

        oldest

        votes









        2












        $begingroup$

        If Lagrange's is not mandatory, WLOG choose $x=cos t, y=sin t$ to find



        $$f(x,y)=Acos^2t+Bsin tcos t+Csin^2t=g(t)text{(say)}$$



        Use $cos2t=1-2sin^2t=2cos^2t-1,sin2t=2sin tcos t$ to find $g(t)$ to be of the form $$pcos2t+qsin2t+r$$



        Now use $-sqrt{p^2+q^2}le pcos2t+qsin2tlesqrt{p^2+q^2}$






        share|cite|improve this answer









        $endgroup$













        • $begingroup$
          Thank you very much!!
          $endgroup$
          – hew
          Jan 11 at 6:35
















        2












        $begingroup$

        If Lagrange's is not mandatory, WLOG choose $x=cos t, y=sin t$ to find



        $$f(x,y)=Acos^2t+Bsin tcos t+Csin^2t=g(t)text{(say)}$$



        Use $cos2t=1-2sin^2t=2cos^2t-1,sin2t=2sin tcos t$ to find $g(t)$ to be of the form $$pcos2t+qsin2t+r$$



        Now use $-sqrt{p^2+q^2}le pcos2t+qsin2tlesqrt{p^2+q^2}$






        share|cite|improve this answer









        $endgroup$













        • $begingroup$
          Thank you very much!!
          $endgroup$
          – hew
          Jan 11 at 6:35














        2












        2








        2





        $begingroup$

        If Lagrange's is not mandatory, WLOG choose $x=cos t, y=sin t$ to find



        $$f(x,y)=Acos^2t+Bsin tcos t+Csin^2t=g(t)text{(say)}$$



        Use $cos2t=1-2sin^2t=2cos^2t-1,sin2t=2sin tcos t$ to find $g(t)$ to be of the form $$pcos2t+qsin2t+r$$



        Now use $-sqrt{p^2+q^2}le pcos2t+qsin2tlesqrt{p^2+q^2}$






        share|cite|improve this answer









        $endgroup$



        If Lagrange's is not mandatory, WLOG choose $x=cos t, y=sin t$ to find



        $$f(x,y)=Acos^2t+Bsin tcos t+Csin^2t=g(t)text{(say)}$$



        Use $cos2t=1-2sin^2t=2cos^2t-1,sin2t=2sin tcos t$ to find $g(t)$ to be of the form $$pcos2t+qsin2t+r$$



        Now use $-sqrt{p^2+q^2}le pcos2t+qsin2tlesqrt{p^2+q^2}$







        share|cite|improve this answer












        share|cite|improve this answer



        share|cite|improve this answer










        answered Jan 11 at 6:08









        lab bhattacharjeelab bhattacharjee

        224k15156274




        224k15156274












        • $begingroup$
          Thank you very much!!
          $endgroup$
          – hew
          Jan 11 at 6:35


















        • $begingroup$
          Thank you very much!!
          $endgroup$
          – hew
          Jan 11 at 6:35
















        $begingroup$
        Thank you very much!!
        $endgroup$
        – hew
        Jan 11 at 6:35




        $begingroup$
        Thank you very much!!
        $endgroup$
        – hew
        Jan 11 at 6:35











        1












        $begingroup$

        This is simple enough that you can use your constraint as a substitution, i.e., $y = sqrt{1 - x^2}$.



        Then your function is:



        $$f(x) = left(2 sqrt{1-x^2}+xright)^2+left(4 sqrt{1-x^2}+3 xright)^2$$



        enter image description here



        Its derivative is:



        $${d f(x) over dx} = -frac{4 left(14 x^2+5 sqrt{1-x^2} x-7right)}{sqrt{1-x^2}}$$



        Set it to zero and find:



        $$x = left{-sqrt{frac{1}{2}+frac{5}{2 sqrt{221}}},sqrt{frac{1}{442} left(221-5 sqrt{221}right)}right} approx { -0.817416, 0.576048 }$$



        where clearly one is a minimum, the other a maximum.



        For "culture," here is the figure in three dimensions:



        enter image description here






        share|cite|improve this answer











        $endgroup$













        • $begingroup$
          Thank you very much!!
          $endgroup$
          – hew
          Jan 11 at 6:34










        • $begingroup$
          @hew: So perhaps a (+1) is in order?
          $endgroup$
          – David G. Stork
          Jan 11 at 6:36
















        1












        $begingroup$

        This is simple enough that you can use your constraint as a substitution, i.e., $y = sqrt{1 - x^2}$.



        Then your function is:



        $$f(x) = left(2 sqrt{1-x^2}+xright)^2+left(4 sqrt{1-x^2}+3 xright)^2$$



        enter image description here



        Its derivative is:



        $${d f(x) over dx} = -frac{4 left(14 x^2+5 sqrt{1-x^2} x-7right)}{sqrt{1-x^2}}$$



        Set it to zero and find:



        $$x = left{-sqrt{frac{1}{2}+frac{5}{2 sqrt{221}}},sqrt{frac{1}{442} left(221-5 sqrt{221}right)}right} approx { -0.817416, 0.576048 }$$



        where clearly one is a minimum, the other a maximum.



        For "culture," here is the figure in three dimensions:



        enter image description here






        share|cite|improve this answer











        $endgroup$













        • $begingroup$
          Thank you very much!!
          $endgroup$
          – hew
          Jan 11 at 6:34










        • $begingroup$
          @hew: So perhaps a (+1) is in order?
          $endgroup$
          – David G. Stork
          Jan 11 at 6:36














        1












        1








        1





        $begingroup$

        This is simple enough that you can use your constraint as a substitution, i.e., $y = sqrt{1 - x^2}$.



        Then your function is:



        $$f(x) = left(2 sqrt{1-x^2}+xright)^2+left(4 sqrt{1-x^2}+3 xright)^2$$



        enter image description here



        Its derivative is:



        $${d f(x) over dx} = -frac{4 left(14 x^2+5 sqrt{1-x^2} x-7right)}{sqrt{1-x^2}}$$



        Set it to zero and find:



        $$x = left{-sqrt{frac{1}{2}+frac{5}{2 sqrt{221}}},sqrt{frac{1}{442} left(221-5 sqrt{221}right)}right} approx { -0.817416, 0.576048 }$$



        where clearly one is a minimum, the other a maximum.



        For "culture," here is the figure in three dimensions:



        enter image description here






        share|cite|improve this answer











        $endgroup$



        This is simple enough that you can use your constraint as a substitution, i.e., $y = sqrt{1 - x^2}$.



        Then your function is:



        $$f(x) = left(2 sqrt{1-x^2}+xright)^2+left(4 sqrt{1-x^2}+3 xright)^2$$



        enter image description here



        Its derivative is:



        $${d f(x) over dx} = -frac{4 left(14 x^2+5 sqrt{1-x^2} x-7right)}{sqrt{1-x^2}}$$



        Set it to zero and find:



        $$x = left{-sqrt{frac{1}{2}+frac{5}{2 sqrt{221}}},sqrt{frac{1}{442} left(221-5 sqrt{221}right)}right} approx { -0.817416, 0.576048 }$$



        where clearly one is a minimum, the other a maximum.



        For "culture," here is the figure in three dimensions:



        enter image description here







        share|cite|improve this answer














        share|cite|improve this answer



        share|cite|improve this answer








        edited Jan 11 at 19:19

























        answered Jan 11 at 6:05









        David G. StorkDavid G. Stork

        10.7k31332




        10.7k31332












        • $begingroup$
          Thank you very much!!
          $endgroup$
          – hew
          Jan 11 at 6:34










        • $begingroup$
          @hew: So perhaps a (+1) is in order?
          $endgroup$
          – David G. Stork
          Jan 11 at 6:36


















        • $begingroup$
          Thank you very much!!
          $endgroup$
          – hew
          Jan 11 at 6:34










        • $begingroup$
          @hew: So perhaps a (+1) is in order?
          $endgroup$
          – David G. Stork
          Jan 11 at 6:36
















        $begingroup$
        Thank you very much!!
        $endgroup$
        – hew
        Jan 11 at 6:34




        $begingroup$
        Thank you very much!!
        $endgroup$
        – hew
        Jan 11 at 6:34












        $begingroup$
        @hew: So perhaps a (+1) is in order?
        $endgroup$
        – David G. Stork
        Jan 11 at 6:36




        $begingroup$
        @hew: So perhaps a (+1) is in order?
        $endgroup$
        – David G. Stork
        Jan 11 at 6:36











        0












        $begingroup$

        Exact solution with Lagrange multiplier method is:
        $$f_{min}=15-sqrt{221}approx0.134$$
        if
        $$x=frac{sqrt{5 sqrt{221}+221}}{sqrt{442}}approx0.817,;
        y=-frac{sqrt{221-5 sqrt{221}}}{sqrt{442}}approx-0.576$$

        or
        $$x=-frac{sqrt{5 sqrt{221}+221}}{sqrt{442}}approx-0.817,;
        y=frac{sqrt{221-5 sqrt{221}}}{sqrt{442}}approx0.576$$

        enter image description here






        share|cite|improve this answer









        $endgroup$


















          0












          $begingroup$

          Exact solution with Lagrange multiplier method is:
          $$f_{min}=15-sqrt{221}approx0.134$$
          if
          $$x=frac{sqrt{5 sqrt{221}+221}}{sqrt{442}}approx0.817,;
          y=-frac{sqrt{221-5 sqrt{221}}}{sqrt{442}}approx-0.576$$

          or
          $$x=-frac{sqrt{5 sqrt{221}+221}}{sqrt{442}}approx-0.817,;
          y=frac{sqrt{221-5 sqrt{221}}}{sqrt{442}}approx0.576$$

          enter image description here






          share|cite|improve this answer









          $endgroup$
















            0












            0








            0





            $begingroup$

            Exact solution with Lagrange multiplier method is:
            $$f_{min}=15-sqrt{221}approx0.134$$
            if
            $$x=frac{sqrt{5 sqrt{221}+221}}{sqrt{442}}approx0.817,;
            y=-frac{sqrt{221-5 sqrt{221}}}{sqrt{442}}approx-0.576$$

            or
            $$x=-frac{sqrt{5 sqrt{221}+221}}{sqrt{442}}approx-0.817,;
            y=frac{sqrt{221-5 sqrt{221}}}{sqrt{442}}approx0.576$$

            enter image description here






            share|cite|improve this answer









            $endgroup$



            Exact solution with Lagrange multiplier method is:
            $$f_{min}=15-sqrt{221}approx0.134$$
            if
            $$x=frac{sqrt{5 sqrt{221}+221}}{sqrt{442}}approx0.817,;
            y=-frac{sqrt{221-5 sqrt{221}}}{sqrt{442}}approx-0.576$$

            or
            $$x=-frac{sqrt{5 sqrt{221}+221}}{sqrt{442}}approx-0.817,;
            y=frac{sqrt{221-5 sqrt{221}}}{sqrt{442}}approx0.576$$

            enter image description here







            share|cite|improve this answer












            share|cite|improve this answer



            share|cite|improve this answer










            answered Jan 11 at 8:11









            Aleksas DomarkasAleksas Domarkas

            8976




            8976























                0












                $begingroup$

                It is possible to find the solution by using the polar coordinates:



                $$f(theta) = 10cos^2theta + 20sin^2theta + 28sinthetacostheta = 10 + 10sin^2theta + 14sin2theta$$



                The extrema are found by deriving:



                $$f'(theta) = 20sintheta costheta + 28cos2theta = 10sin2theta + 28cos2theta = 0$$



                And we get
                $$theta = -frac{1}{2} arctanfrac{14}{5} + kfrac{pi}{2} $$
                where $k$ is any integer between $0$ and $3$



                We can check numerically that the maximum corresponds to
                $$theta = -frac{1}{2} arctanfrac{14}{5} + kpi approx -0.6138 + kpi$$



                where $k$ is equal to $0$ or $1$



                This corresponds to $x approx 0.81754, ,y approx -0.576$ and $x approx -0.81754, ,y approx 0.576$






                share|cite|improve this answer











                $endgroup$


















                  0












                  $begingroup$

                  It is possible to find the solution by using the polar coordinates:



                  $$f(theta) = 10cos^2theta + 20sin^2theta + 28sinthetacostheta = 10 + 10sin^2theta + 14sin2theta$$



                  The extrema are found by deriving:



                  $$f'(theta) = 20sintheta costheta + 28cos2theta = 10sin2theta + 28cos2theta = 0$$



                  And we get
                  $$theta = -frac{1}{2} arctanfrac{14}{5} + kfrac{pi}{2} $$
                  where $k$ is any integer between $0$ and $3$



                  We can check numerically that the maximum corresponds to
                  $$theta = -frac{1}{2} arctanfrac{14}{5} + kpi approx -0.6138 + kpi$$



                  where $k$ is equal to $0$ or $1$



                  This corresponds to $x approx 0.81754, ,y approx -0.576$ and $x approx -0.81754, ,y approx 0.576$






                  share|cite|improve this answer











                  $endgroup$
















                    0












                    0








                    0





                    $begingroup$

                    It is possible to find the solution by using the polar coordinates:



                    $$f(theta) = 10cos^2theta + 20sin^2theta + 28sinthetacostheta = 10 + 10sin^2theta + 14sin2theta$$



                    The extrema are found by deriving:



                    $$f'(theta) = 20sintheta costheta + 28cos2theta = 10sin2theta + 28cos2theta = 0$$



                    And we get
                    $$theta = -frac{1}{2} arctanfrac{14}{5} + kfrac{pi}{2} $$
                    where $k$ is any integer between $0$ and $3$



                    We can check numerically that the maximum corresponds to
                    $$theta = -frac{1}{2} arctanfrac{14}{5} + kpi approx -0.6138 + kpi$$



                    where $k$ is equal to $0$ or $1$



                    This corresponds to $x approx 0.81754, ,y approx -0.576$ and $x approx -0.81754, ,y approx 0.576$






                    share|cite|improve this answer











                    $endgroup$



                    It is possible to find the solution by using the polar coordinates:



                    $$f(theta) = 10cos^2theta + 20sin^2theta + 28sinthetacostheta = 10 + 10sin^2theta + 14sin2theta$$



                    The extrema are found by deriving:



                    $$f'(theta) = 20sintheta costheta + 28cos2theta = 10sin2theta + 28cos2theta = 0$$



                    And we get
                    $$theta = -frac{1}{2} arctanfrac{14}{5} + kfrac{pi}{2} $$
                    where $k$ is any integer between $0$ and $3$



                    We can check numerically that the maximum corresponds to
                    $$theta = -frac{1}{2} arctanfrac{14}{5} + kpi approx -0.6138 + kpi$$



                    where $k$ is equal to $0$ or $1$



                    This corresponds to $x approx 0.81754, ,y approx -0.576$ and $x approx -0.81754, ,y approx 0.576$







                    share|cite|improve this answer














                    share|cite|improve this answer



                    share|cite|improve this answer








                    edited Jan 11 at 9:37

























                    answered Jan 11 at 9:18









                    DamienDamien

                    60714




                    60714























                        0












                        $begingroup$

                        The maximum value of a quadratic form on the unit circle is equal to its largest eigenvalue. $f$ simplifies to $10x^2+28xy+20y^2$ and so the eigenvalues of $f$ are the roots of $lambda^2-30lambda+4$.






                        share|cite|improve this answer









                        $endgroup$


















                          0












                          $begingroup$

                          The maximum value of a quadratic form on the unit circle is equal to its largest eigenvalue. $f$ simplifies to $10x^2+28xy+20y^2$ and so the eigenvalues of $f$ are the roots of $lambda^2-30lambda+4$.






                          share|cite|improve this answer









                          $endgroup$
















                            0












                            0








                            0





                            $begingroup$

                            The maximum value of a quadratic form on the unit circle is equal to its largest eigenvalue. $f$ simplifies to $10x^2+28xy+20y^2$ and so the eigenvalues of $f$ are the roots of $lambda^2-30lambda+4$.






                            share|cite|improve this answer









                            $endgroup$



                            The maximum value of a quadratic form on the unit circle is equal to its largest eigenvalue. $f$ simplifies to $10x^2+28xy+20y^2$ and so the eigenvalues of $f$ are the roots of $lambda^2-30lambda+4$.







                            share|cite|improve this answer












                            share|cite|improve this answer



                            share|cite|improve this answer










                            answered Jan 11 at 9:53









                            amdamd

                            29.7k21050




                            29.7k21050






























                                draft saved

                                draft discarded




















































                                Thanks for contributing an answer to Mathematics Stack Exchange!


                                • Please be sure to answer the question. Provide details and share your research!

                                But avoid



                                • Asking for help, clarification, or responding to other answers.

                                • Making statements based on opinion; back them up with references or personal experience.


                                Use MathJax to format equations. MathJax reference.


                                To learn more, see our tips on writing great answers.




                                draft saved


                                draft discarded














                                StackExchange.ready(
                                function () {
                                StackExchange.openid.initPostLogin('.new-post-login', 'https%3a%2f%2fmath.stackexchange.com%2fquestions%2f3069516%2ffind-maximum-value-under-constraints-by-lagrange-multipliers-or-another-method%23new-answer', 'question_page');
                                }
                                );

                                Post as a guest















                                Required, but never shown





















































                                Required, but never shown














                                Required, but never shown












                                Required, but never shown







                                Required, but never shown

































                                Required, but never shown














                                Required, but never shown












                                Required, but never shown







                                Required, but never shown







                                Popular posts from this blog

                                Mario Kart Wii

                                What does “Dominus providebit” mean?

                                Antonio Litta Visconti Arese